LSAT and Law School Admissions Forum

Get expert LSAT preparation and law school admissions advice from PowerScore Test Preparation.

User avatar
 Dave Killoran
PowerScore Staff
  • PowerScore Staff
  • Posts: 5849
  • Joined: Mar 25, 2011
|
#43443
Complete Question Explanation
(The complete setup for this game can be found here: lsat/viewtopic.php?t=16406)

The correct answer choice is (E)

If L and M are in group 2, then G must be in group 1 (because of M), and H must be in group 2 (because of the contrapositive of the third rule). With H, L, and M in group 2, there is not enough room for the FJ block, and thus we can infer that the FJ block is in group 1. Only N and K remain, and each variable could be in group 1 or 2.
D94_Game_#1_#4_diagram 1.png
N is not among the choices, but K is, and hence answer choice (E) is correct.

Get the most out of your LSAT Prep Plus subscription.

Analyze and track your performance with our Testing and Analytics Package.